You are on page 1of 43

PRE-SAMIKSHA 2023

ALL INDIA OPEN MOCK


TEST – 2

GENERAL STUDIES
PAPER-I

SOLUTION

Office Complex 6, 3rd Floor, Old Rajinder Nagar New Delhi 110060|
Ph: +91-8826486658, +918826496658, | Email: students@levelupias.com
Office Complex 6, 3rd Floor, Old Rajinder Nagar New Delhi 110060|
Ph: +91-8826486658, +918826496658, | Email: students@levelupias.com
Q1.
Answer: a
Explanation:
• Gross Domestic Product is the total market value of all final goods and services
produced in the economy in one year.
• Real Gross Domestic Product (GDP) refers to the current year's production of goods
and services valued at base year prices. An increase in the quantity of output
produced in an economy would lead to an increase in the total value of goods and
services produced and hence an increase in real GDP.
• While calculating the real GDP, the quantity of output produced in a specific year
would be multiplied by the base year prices. Therefore, increasing the quantity of
output produced in a year would result in an increased real GDP. It is used to
compute economic growth. The percentage change in real GDP is the GDP growth
rate. So, Option (a) is correct.

Q2.
Answer: a
Explanation:
• Higher interest rates in the US could lead to a decline in capital flows to India. Indian
companies may face higher borrowing costs due to higher interest rates in the US. But it
is not necessary that this will lead to immediate slowdown in the economy. So,
statement 1 is incorrect.
• When the US Federal Reserve increases interest rates, the cost of borrowing in the US
goes up. As a result, the Indian government may have to pay higher interest rates on its
foreign borrowings, which could lead to a higher cost of debt for the government. So,
statement 2 is correct.
• Higher interest rates in the US could lead to a decline in the stock prices of Indian
companies, as foreign investors may prefer to invest in the US instead of India. This could
lead to a sell-off in Indian stocks, which could cause the stock market to decline. So,
statement 3 is correct.
• Higher interest rates in the US could lead to a strengthening of the US dollar against
other currencies, including the Indian rupee. This could lead to a depreciation of the
Indian rupee against the US dollar, and not an appreciation as stated in statement 4. So,
statement 4 is incorrect

Q3.
Answer: b
Explanation:
RBI triggers the Prompt Corrective Action when banks breach certain regulatory
requirements like minimum capital, quantum of non-performing assets etc.
Previously, the trigger points were,
o Capital to Risk-weighted Assets Ratio
o Non-Performing Assets of the bank (NPA)
o Leverage or Total debt level
o Profitability- Return on asset

Office Complex 6, 3rd Floor, Old Rajinder Nagar New Delhi 110060|
Ph: +91-8826486658, +918826496658, | Email: students@levelupias.com
According to RBI's new guidelines, Capital, Asset quality, and Leverage will be the three key
areas for monitoring in the revised framework. RBI removed Profitability as the criteria for
the regulations under PCA. So, the trigger point 4 is incorrect.
NOTE:
o Reserve Bank of India introduced a PCA for Scheduled Commercial Banks in 2002.
· RBI brought “supervisory action framework (SAF)” for UCBs in a place of PCA for
commercial banks.

Q4.
Answer: c
EXPLANATION:
· Supply-side inflation is defined as because of less supply, the overall price level increases
due to higher production costs, which reflects increased prices of goods and commodities.
· Releasing buffer stocks into the market: This measure can increase the supply of essential
goods and help stabilize prices. So, Option (a) is not correct.
· Taking strict action against hoarding of goods: Hoarding of goods can lead to an artificial
scarcity of goods and drive-up prices. Therefore, taking strict action against hoarding can
help increase the supply of goods and stabilize prices. So, Option (b) is not correct.
· Raising indirect tax: This measure is likely to be counterproductive during times of high
inflation caused by supply-side bottlenecks. Indirect taxes such as excise duty, customs duty,
and service tax are added to the cost of goods and services, making them more expensive
for consumers. This can lead to a further increase in prices, exacerbating inflation. So,
Option (c) is correct.
· Waiving of import duty on items in short supply: This measure can help increase the supply
of goods in the domestic market and help stabilize prices. So, Option (d) is not correct.

Q5.
Answer: d
Explanation:
• Transfer pricing refers to the prices of goods and services that are exchanged between
companies under common control. For example, if a subsidiary company sells goods or
renders services to its holding company or a sister company, the price charged is referred to
as the transfer price. Transfer pricing leads to tax savings for organizations, and companies
widely use this practice to reduce to burden of tax on them. So, Statement 1 is correct.
• Round tripping is a practice where funds are transferred from one country to another and
transferred back to the origin country for purposes like money laundering or to get the
benefit of tax concession/evasion/avoidance from countries like Mauritius which enjoy low
taxes etc. Round tripping is when funds flow from a country to a foreign country and flows
back to the same country in the form of foreign investment. However, the term “Round
Tripping” is not defined under Foreign Exchange Management Act, 1999. This phenomenon
takes place due to many reasons, and the most common of them is for tax
avoidance/evasion or concession purposes. So, Statement 2 is correct.
BEPS refers to tax planning strategies that exploit gaps and mismatches in tax rules to
artificially shift profits to low or no-tax locations where there is little or no economic activity
or to erode tax bases through deductible payments such as interest or royalties. When that
income / profit is transferred to another country or tax haven, the tax base is eroded and

Office Complex 6, 3rd Floor, Old Rajinder Nagar New Delhi 110060|
Ph: +91-8826486658, +918826496658, | Email: students@levelupias.com
the company does not pay taxes to the country that is generating the income. As a result,
tax revenues are reduced and the government is detained. So, Statement 3 is correct.

Q6.
Answer: c
Explanation:
• The Peace Clause is a provision in the WTO's Agreement on Agriculture that grants
developing countries a temporary exemption from certain rules on agricultural subsidies.
Under the Peace Clause, developing countries are allowed to provide subsidies that are
above the limits set by the WTO, if those subsidies do not distort trade or have a negative
effect on other WTO members. So, statement 1 is correct.
• The Peace Clause was first introduced in the Uruguay Round of negotiations and was
subsequently extended in the 2013 Bali Ministerial Conference. The latest extension at the
2013 Bali Ministerial Conference is valid until a permanent solution is found. So, statement
2 is correct.
• The de minimis level is a key component of the Peace Clause of the World Trade
Organization's (WTO) Agreement on Agriculture. The de minimis level is the maximum
amount of subsidies that a developing country can provide for a particular agricultural
product without triggering a challenge from other WTO members. The de minimis level for
developing countries that are not members of the Organization for Economic Cooperation
and Development (OECD), the de minimis level is set at 10% of the value of the agricultural
product. For developing countries that are members of the OECD, the de minimis level is set
at 5% of the value of the agricultural product. So, statement 3 is not correct.
• Recently, India’s MSP programmes are under scrutiny at the WTO, as it is the first country to
invoke the Bali ‘peace clause’ to justify exceeding its 10% ceiling (of the total value of rice
production) for rice support in 2018-2019 and 2019-2020.
• Article 13 (“due restraint”) of the Agriculture Agreement protects countries using subsidies
which comply with the agreement from being challenged under other WTO agreements.
Without this “peace clause”, countries would have greater freedom to take action against
each others subsidies, under the Subsidies and Countervailing Measures Agreement and
related provisions.

Q7.
Answer: a
Explanation:
• Pump priming refers to the process of increasing government spending to boost economic
activity.
• Pump priming, in economics, refers to a set of government policies or actions designed to
stimulate economic growth and increase demand during a period of economic downturn.
This is typically done through an increase in government spending or a decrease in taxes,
which can lead to an increase in consumer spending and business investment.
• The idea behind pump priming is that during times of economic recession or stagnation,
businesses and consumers are hesitant to spend money, leading to a decrease in overall
economic activity. By increasing government spending or decreasing taxes, the government
can inject money into the economy, increasing demand and encouraging businesses and
consumers to spend more.

Office Complex 6, 3rd Floor, Old Rajinder Nagar New Delhi 110060|
Ph: +91-8826486658, +918826496658, | Email: students@levelupias.com
• While pump priming can be effective in stimulating economic growth, it can also lead to
higher government debt and inflation. Additionally, the effectiveness of pump priming can
depend on a variety of factors, including the overall health of the economy, the specific
policies implemented, and the timing and duration of those policies.

Q8.
Answer: c
EXPLANATION:
· Financial systems can be classified into two categories based on the maturity period. The
money market usually deals with short-term funds with a one-year maturity period,
whereas the capital market deals with long-term funds with a maturity period of more than
a year. So, Option (a) is correct.
· Dated Government Securities(G-Secs) are capital market instruments with a tenor that
ranges from 5 years to 40 years. In India, the Central Government issues treasury bills and
bonds or dated securities, while the State Governments issue only bonds or dated securities,
called the State Development Loans (SDLs). State Governments cannot issue T-Bills. Thus,
Treasury bills are money market instruments, while dated government securities are capital
market instruments. So, Option (b) is correct.
· Financial markets comprise money and capital markets in which the Reserve Bank of India
regulates the organized money market. Indian Capital Markets are regulated and monitored
by the Ministry of Finance, The Securities and Exchange Board of India and The Reserve
Bank of India. providing efficient legislative framework for securities markets. So, Option (c)
is not correct.
Shares are financial instruments usually transacted in the capital market, not the money
market, as it involves financing long-term capital requirements. So, Option (d) is correct.

Q9.
Answer: b
Explanation:
• The excess of total expenditure over total receipts excluding borrowings is called Fiscal
Deficit. In other words, the Fiscal Deficit gives the amount needed by the government to
meet its expenses. Thus, a large Fiscal Deficit means a large number of borrowings. Fiscal
Deficit includes Interest component, Only Primary deficit excludes Interest. So, Statement 1
is incorrect.
• Primary deficit indicates the borrowing requirements of the govt. for meeting expenditure
excluding interest payment. It is calculated by, Primary deficit = Fiscal deficit – Interest
payments. When the primary deficit is zero, the fiscal deficit becomes equal to the interest
payment. Entire borrowings of the Government are used to make interest payments.
Therefore, Zero Primary deficit doesn’t indicate that the interest payment on the loans
taken by the government is zero. So, Statement 2 is incorrect.
Effective Revenue Deficit is the difference between revenue deficit and grants for creation
of capital assets. Formula: Effective Revenue Deficit = Revenue Deficit - Grants in aid for
capital assets. Effective Revenue Deficit was introduced in the Union Budget of 2011-12. So,
Statement 3 is correct.

Office Complex 6, 3rd Floor, Old Rajinder Nagar New Delhi 110060|
Ph: +91-8826486658, +918826496658, | Email: students@levelupias.com
Q10.
Answer: c
Explanation:
Both Statements are correct:
A liquidity adjustment facility (LAF) is a tool used in monetary policy, mainly by the Reserve
Bank of India (RBI), which enables banks to borrow money through repurchase agreements
(reposals) or banks to lend to the RBI using reverse repo contracts.
In April 2022, Monetary Policy With this fixed rate overnight reverse repo has ceased to be
the floor of the LAF corridor.
SDF, as it stands currently has the following features:
▫ It is floor of the LAF corridor, replacing the hitherto fixed rate reverse repo.
▫ It is a monetary policy instrument to absorb liquidity without collateral (collaterals in this
case are normally government securities) with an interest rate of 3.75% (25 bps below the
repo rate)
▫ It is operated on overnight basis, with the flexibility to absorb liquidity for longer tenor
with appropriate pricing.
▫ Deposits under SDF shall not be reckoned as balances eligible for the maintenance of the
CRR under section 42 of the RBI Act, 1934, but shall be an eligible asset for maintenance of
the SLR under section 24 of the Banking Regulation Act, 1949.

Q11.
Answer: d
Explanation:
A five-judge Constitution Bench of the Supreme Court unanimously ruled that a high-power
committee for the appointment of the Chief Election Commissioner (CEC) and Election
Commissioners (ECs).
• They will be appointed on the advice of a committee comprising the Prime Minister, the
Leader of Opposition in Lok Sabha, and the Chief Justice of India. Hence, statement 1 is
incorrect.
• The election commissioners and the chief election commissioner of India enjoys same
salary, privilege but not the security of tenure which has been given to a judge of supreme
court. Election commissioners can be removed by president on the advice of chief election
commissioner. Hence, statement 2 is incorrect.

Q12.
Answer: d
Explanation:

• Under Article 159, every Governor and every person discharging the functions of the
Governor shall, before entering upon his office, make and subscribe in the presence
of the Chief Justice of the High Court exercising jurisdiction in relation to the State, or,
in his absence, the seniormost Judge of that Court available. Hence, A is correct.
• Article 158 (4) provides that salaries of governor shall not be diminished during term
of his office. Hence, B is correct.
• A Governor shall hold office for a term of five years from the date on which he enters
upon his office however his tenure is dependent upon the pleasure of the president.
Hence, C is correct.

Office Complex 6, 3rd Floor, Old Rajinder Nagar New Delhi 110060|
Ph: +91-8826486658, +918826496658, | Email: students@levelupias.com
• Under Article 158 (3A) Where the same person is appointed as Governor of two or
more States, the emoluments and allowances payable to the Governor shall be
allocated among the States in such proportion as the President may by order
determine. Hence, D is incorrect.

Q13.
Answer: c
Explanation:
• Election of Vice President is done through a system of proportional representation
through single transferable vote using secret ballot. Hence, statement 1 is correct and
statement 2 is incorrect.
• Vice president is elected by an electoral college which shall consists of all the members
of parliament including elected as well as nominated members.
• All disputes arising out of election of Vice President shall be decided by the Supreme
Court of India. Hence, statement 3 is incorrect.

Q14.
Answer: b
Explanation:
Statements 1 and 2 are correct.
All authorities, civil and judicial, in the territory of India shall act in aid of the Supreme Court.
(Article 144)
The minimum number of judges required to sit for hearing a case on which reference has
been made to the supreme court by the president is five. (Article 145(3))
Article 145(5) provides that all the judgements of supreme court shall be taken by a majority
of the bench but this should not stop any judge on the bench to deliver any dissenting
judgment from that of bench. Hence statement 3 is not correct.

Q15.
Answer: c
Explanation:

• The Representation of People Act 1951 does not provide for disqualification of
members of the legislature with retrospective effect. This means that a person who
was not disqualified at the time of their election, but subsequently becomes
disqualified due to a change in the law or a court decision, cannot be removed from
their position as a member of the legislature for actions taken before the
disqualification came into effect. Hence, A is incorrect
• A person who is disqualified from contesting election on grounds of conviction of more
than two years then he is also disqualified from voting. Hence, B is incorrect
• A person shall be disqualified if, and for so long as, there subsists a contract entered
into by him in the course of his trade or business with the appropriate Government
for the supply of goods to, or for the execution of any works undertaken by, that
Government. Hence, C is correct.

Office Complex 6, 3rd Floor, Old Rajinder Nagar New Delhi 110060|
Ph: +91-8826486658, +918826496658, | Email: students@levelupias.com
• Section 11 of the act provides that The Election Commission may, for reasons to be
recorded, remove any disqualification except under section 8A or reduce the period
of any such disqualification. Hence option D is incorrect.

Q16.
Answer: a
Explanation:
• Our Constitution reinforces and reinvents forms of liberal individualism. This is an
important achievement because this is done in the backdrop of a society where
community values are often indifferent or hostile to individual autonomy.
• Constitution upholds the principle of social justice without compromising on individual
liberties. The constitutional commitment to caste-based affirmative action
programme shows how much ahead India was compared to other nations.
• India strives to retain regional identities along with the national identity. The
Constitution constantly reinforces a common national identity. The Indian
Constitution tried to balance these various identities. Yet, preference was given to
common identity under certain conditions. This is clarified in the debate over separate
electorates based on religious identity which the Constitution rejects. Separate
electorates were rejected not because they fostered difference between religious
communities as such or because they endangered a simple notion of national unity.
Hence, statement 3 is incorrect

Q17.
Answer: c
Explanation:
• Article 249 prescribes the Power of Parliament to legislate with respect to a matter in
the State List in the national interest. It says if the Council of States has declared by
resolution supported by not less than two-thirds of the members present and voting
that it is necessary or expedient in the national interest that Parliament should make
laws with respect any matter enumerated in the State List specified in the resolution,
the parliament can enact laws on such matters. Hence, statement 1 is correct.
• A resolution passed under this clause shall remain in force for such period not
exceeding one year and it can be extended for a period of another 1 year. Also, law
made under this article cease to have effect on the expiration of a period of six months
after the resolution has ceased to be in force, except as respects things done or
omitted to be done before the expiration of the said period. Hence, statement 2 is
correct.

Q18.
Ans: c
Explanation:
Article 279A of the constitution which was added by the 101st Amendment act provides for
the composition of the GST council.
The Goods and Services Tax Council shall consist of the following members, namely:
(a) the Union Finance Minister — Chairperson;

Office Complex 6, 3rd Floor, Old Rajinder Nagar New Delhi 110060|
Ph: +91-8826486658, +918826496658, | Email: students@levelupias.com
(b) the Union Minister of State in charge of Revenue or Finance — Member;
(c) the Minister in charge of Finance or Taxation or any other Minister nominated by each
State Government — Members

• One-half of the total number of Members of the Goods and Services Tax Council shall
constitute the quorum at its meetings. Hence statement 1 is correct.
• Every decision of the Goods and Services Tax Council shall be taken at a meeting, by a
majority of not less than three-fourths of the weighted votes of the members present
and voting, in accordance with the following principles, namely:
(a) the vote of the Central Government shall have a weightage of one-third of the total
votes cast, and
(b) the votes of all the State Governments taken together shall have a weightage of
two-thirds of the total votes cast, in that meeting. Hence statement 2 is not correct.

• Article 279A (11) Provides that The Goods and Services Tax Council shall establish a
mechanism to adjudicate any dispute:
(a) between the Government of India and one or more States; or
(b) between the Government of India and any State or States on one side and one or
more other States on the other side; or
(c) between two or more States, arising out of the recommendations of the Council or
implementation thereof. Hence statement 3 is correct.

Q19.
Answer: d
Explanation:
Statement 1 is incorrect: Every national party and every state party is allotted a symbol
exclusively reserved for its use throughout the country and the states respectively. But a
registered unrecognised party is not allotted symbol.
Statement 2 is incorrect: The Election Symbols (Reservation and Allotment) Order, 1968
empowers the Election Commission to recognize political parties and allot symbols.
It does so in exercise of the powers conferred by Article 324 of the Constitution, read with
section 29A of the Representation of the People Act, 1951 (43 of 1951) and rules 5 and 10 of
the Conduct of Elections Rules, 1961 and all other powers enabling it in this behalf.
Statement 3 is incorrect: The splinter group of the party one group gets the symbol of the
original party which represents majority of the party- other than the group that got the
party symbol - had to register itself as a separate party. They could lay claim to national or
state party status only on the basis of its performance in state or central elections after
registration.

Q20.
Answer: d
Explanation: The term sovereign in the preamble to Indian constitution is inspired from
Article 5 of the Irish Constitution which states that Ireland is a sovereign, independent,
democratic state.
Sovereignty is a key concept in political theory. It constitutes one of the four elements of the
state without which statehood remains incomplete. Derived from the Latin term Superanus,

Office Complex 6, 3rd Floor, Old Rajinder Nagar New Delhi 110060|
Ph: +91-8826486658, +918826496658, | Email: students@levelupias.com
which means supreme, sovereignty denotes the supreme power of the state to extract
obedience from the people who inhabit it. It means that the power of the state is
unquestionable, and the state has a right to demand allegiance from its citizens. It also
means that the violation of the command of the state would invite penalties or other
punishment.

Q21.
Answer: b
Explanation:
Biosparging is an in-situ remediation technology that uses indigenous microorganisms to
biodegrade organic constituents in the saturated zone. In biosparging, air (oxygen) and
nutrients (if needed) are injected into the saturated zone to increase the biological activity
of the indigenous micro-organisms.

Q22.
Answer: c
Explanation:
Dibru-Saikhowa National Park is located in Assam. It was designated a biosphere reserve in
1997. The park is bounded by the Brahmaputra and Lohit Rivers in the north and Dibru river
in the south. It mainly consists of moist mixed semi-evergreen forests, moist deciduous
forests, canebrakes and grasslands. Originally created to help conserve the habitat of the
rare white winged wooded duck, the park is home to other rare creatures such as water
buffalo, black-breasted parrot bill, tiger and capped langur. It is the largest Salix swamp
forest in the north-eastern India, with a tropical monsoon climate with a hot and wet
summer and cool and usually dry winter.

Q23.
Answer: d
Explanation:
• A hydroponic system, often known as soil-less farming, is a method of growing plants
without the use of soil. Plant roots develop in a liquid fertilizer solution or inside
most inert materials such as Rockwool and Vermiculite in this arrangement.
Crops/plants are frequently grown on water, which is rich in critical nutrients, in this
manner.
• Plants cultivated hydroponically develop faster and healthier than plants grown in
the soil, according to research, because essential nutrients are delivered straight to
their roots through the water.
• There is higher productivity than the traditionally farming system, the plant nutrients
are effectively used, and there is no wastage. It provides a controlled climate system
in which the crops can grow year-round.
• The water utilisation very high when compared to the traditional farming method,
up to 80-90% higher but this method also eliminates the attack of pests and insects.

Office Complex 6, 3rd Floor, Old Rajinder Nagar New Delhi 110060|
Ph: +91-8826486658, +918826496658, | Email: students@levelupias.com
Q24.
Answer: b
Explanation:
Pair 1 is correctly matched: Dandeli Elephant Reserve (ER) is located in the Western Ghats
of Karnataka state in India. It is spread over an area of 475 km2 and is known for its diverse
wildlife, including elephants, tigers, leopards, black panthers, and various species of birds.
Pair 2 is incorrectly matched: Singphan Elephant Reserve is located in Nagaland. An
important reserve in North Eastern States, it has huge tracts of forest, strategically located
in contiguity with the Abhaypur Reserve Forest of Assam which is frequented by the moving
herds of elephants.
Pair 3 is incorrectly matched: The Lemru Elephant Reserve is a protected area located in the
central Indian state of Chhattisgarh. The reserve is situated in the northern part of
Chhattisgarh and is bordered by the Mahanadi River to the north and the Kanker district to
the south.
Pair 4 is correctly matched: Recently, Union Ministry of Forest and Environment has given
its approval to the Terai Elephant Reserve (TER) in Uttar Pradesh which covers 3,049 sq km
including Dudhwa Tiger Reserve and Pilibhit Tiger Reserve located in Lakhimpur and Pilibhit
districts.

Q25.
Answer: c
Explanation:
After photosynthesis plants release up to 40% of these carbon rich compounds into the soil
as its exudates to feed fungi and bacteria. These carbon rich atoms do so in order to
promote growth of these organisms (fungi and bacteria), as this will be eventually result in
return of nutrients to plants via nutrient cycle.
Soil Carbon Sequestration:
Statement 1 is correct: Atmospheric carbon is absorbed by plants and pumped into the soil,
where it is stored for hundreds of years by fungal hyphae. This process is known as soil carbon
sequestration. The majority of the fungus consists of hyphae long, narrow tube-like structures
that are mostly of carbon.
Statement 2 is correct: Carbon sequestration activities have been supported through the
CDM (Clean Development Mechanism) under the Kyoto protocol with a focus on afforestation
and reforestation, seen as being the most effective and readily measurable means to
sequester carbon as biomass both above and below ground. In the post-Kyoto negotiations
efforts are being made to give due attention to the huge carbon sequestration potentials in
rangelands.

Q26.
Answer: A
Explanation:
Option A is incorrect: Rig Vedic people did not worship gods for their spiritual uplift or for
ending the miseries of existence. They asked mainly for Praja (Children), Pashu (cattle), food,
wealth, health etc.
Option B is correct: The dominant mode of worshipping the gods was through the recitation
of prayers and offering of sacrifices. Prayers played an important part in Rig Vedic times and
both collective and individual prayers were made.

Office Complex 6, 3rd Floor, Old Rajinder Nagar New Delhi 110060|
Ph: +91-8826486658, +918826496658, | Email: students@levelupias.com
Option C is correct: In Rigveda, various gods are mentioned – Indra, Varuna, Agni, Soma, etc.
However, it also talks about Ekam Sat ViprahaBahudhaVadanti (God is One, but the ways of
reaching could be many). The religion of Rig Vedic Aryans washenotheism which means belief
in single gods, one of them being lauded as the greatest on one occasion and another
receiving the same honour on a different occasion. Moreover, the descriptions of Agni, as
mentioned in Rigveda, indicate towards its omnipresence, thus indicating elements of
monotheism.
Option D is correct: Rig Vedic religion was primarily personification of the forces of nature.
When the Aryans failed to explain the natural happenings around them, they gave these
natural events the names of Gods. For example, Agni, Usha, Ratri, Varuna etc.

Q27.
Answer: C
Explanation:
Statement 1 is correct: Pavaranawas a ceremony in Buddhism wherein the monks used to
confess to the crimes committed by them. It marks the end of the 3 lunar months of Vassa,
sometimes called "Buddhist Lent." The day is marked in some Asian countries where
Theravada Buddhism is practiced.
Statement 2 is correct: Philosophy of Patica-Samuppada, according to which there in chain of
twelve causes and one produces another, the first being ignorance and the twelfth cycle of
birth and death. According to it the root cause of all desires is ajnana (Ignorance).

Q28.
Answer: d
Explanation:
Pair 1 is correctly matched: Ghantasala is a village in Krishna district of Andhra Pradesh. It is
a rare and reputed center for Buddhist sculptures. Kantakasola was the ancient name of
Ghantasala. Alexander Read excavated the Stupa at Ghantasala, which had a circumference
of 112 feet and a height of 23 feet. Ghantasala was once a flourishing town of Indo-Roman
trade as well as important religious centre. The Buddhist relics and the Hindu structures at
the place reveal its past glory. Carved limestone columns belonging to pillared halls associated
with Buddhist monastic establishments (2-3rd centuries BC ) have been discovered.
Pair 2 is correctly matched: Arikamedu was an Indo-Roman trading city and one of the
earliest known Indo-Pacific bead-making centres. The site was mentioned in the Periplus of
the Erythraean Sea an anonymous Greco-Roman text of the 1st century CE which described
the known coastal landmarks and ports of the Indian Subcontinent's coastline.
Pair 3 is correctly matched: Bairat is an ancient Indian City which is presently a famous town.
Virat Nagar was the ancient name of Bairat. In ancient India, it was the capital of a kingdom,
Mahajanapada of Matsya. This kingdom later became a part of Mauryan Empire. Grand stupa
with a circular mound and a circumambulatory path was found here. The ruin of the oldest
Buddhist structure (Chaitya) in India is also found here. One of the minor rock edicts of Ashoka
has been found here. Remains of the earliest free standing religious buildings have been
found here.

Office Complex 6, 3rd Floor, Old Rajinder Nagar New Delhi 110060|
Ph: +91-8826486658, +918826496658, | Email: students@levelupias.com
Q29.
Answer: b
Explanation:
According to Barani, Alauddin Khilji set up three markets at Delhi, first for food-grains, the
second for cloth and expensive items such as sugar, ghee, oil, dry fruits etc., and the third for
horses, slaves and cattle.
Detailed regulations (zawabit) were framed for the control and administration for all these
markets. He issued a set of seven regulations which came to be known as market control
measures. These measures were enacted to regulate the activities of the traders who brought
grain to Delhi. The Sultan fixed the prices of all commodities from grain to cloths, slaves,
cattle, etc.
A controller of market (shahna -i-mandi), intelligence officers(barids) and secret spies
(munhiyan) were appointed. The grain merchants were placedunder the Shahna-i-mandi.

Q30.
Answer: b
Explanation:
Fernao Nunes, also known as FernaoNuniz, was a Portuguese-Jewish traveler, chronicler and
horse trader who spent three years in Vijayanagara, capital of the Vijayanagara Empire in the
time period of Achyuta Deva Raya (1529-42). He lived in Vijayanagar from 1535-1537.
Duarte Barbosa was another Portuguese traveler who visited India in the early 16th century.
He arrived in India in 1500 and spent several years traveling along the west coast of India and
the Malabar Coast. He visited Vijayanagara during the reign of Krishna Deva Raya and wrote
a detailed account of the city and its inhabitants.
Dominigo Paes was a Portuguese traveler who visited India in the early 16th century. He
arrived in India in 1520 and spent several years traveling throughout the country. He visited
Vijayanagara during the reign of Krishna Deva Raya and wrote a detailed account of the city
and its environs. Paes' account is particularly valuable for its descriptions of the military
campaigns and fortifications of the Vijayanagara Empire.
Caesar Frederick an Italian traveller visited Vijayanagar in 1567-68 during the reign of
Sadashiva Raya.

Q31.
Answer: a
Explanation:
Statement 1 is correct: Both Tulsidas (1532-1623) and Surdas (1479-1584) were
contemporary of Akbar (1542-1605).
Tulsidaswas a Ramanandi Vaishnava Hindu saint and poet, renowned for his devotion to the
deity Rama. He wrote several popular works in Sanskrit and Awadhi, but is best known as the
author of the Hanuman Chalisa and of the epic Ramcharitmanas, a retelling of the Sanskrit
Ramayana based on Rama's life in the vernacular Awadhi. His other literary works are Vinaya
Patrika, Geetawali, Dohavali, Sahitya Ratna, Vairagya Sandipani, Janaki Mangal, Parvati
Mangal, and others
Surdas was a formost poet of Krishna sect. He was contemporary of Akbar and Jahangir. He
was a disciple of Vallabhacharya. He preached the religion of love and devotion to a personal
God. He provided inspiration to millions of men and women in Northern India. His works

Office Complex 6, 3rd Floor, Old Rajinder Nagar New Delhi 110060|
Ph: +91-8826486658, +918826496658, | Email: students@levelupias.com
include Sur Sagar, Sahitya Lahiri and Sur Sarawali. Surdas made use of Brajbhasha and the
message that runs through his works is that life itself is poetry.
Statement 2 is incorrect: Gaudiya Vaishnavism (also known as Chaitanya Vaishnavism) is a
Vaishnava Hindu religious movement inspired by Chaitanya Mahaprabhu (1486–1534) in
India. "Gaudiya" refers to the Gaura or Gauḍa region of Bengal, with Vaishnavism meaning
"the worship of Vishnu". Specifically, it is part of Krishnaism—Krishna-centric Vaishnavite
traditions.
Nimbarkacharya, also known as Nimbarka, Nimbaditya or Niyamananda, was a Hindu
philosopher, theologian and the chief proponent of the theology of Dvaitadvaita or dualistic–
non-dualistic. He played a major role in spreading the worship of the divine couple Radha and
Krishna, and founded Nimbarka Sampradaya, one of four main traditions of Hindu sect
Vaishnavism.

Q32.
Answer: b
Explanation:

• Statement 1 is incorrect:
Croatia joined European
union in 2013 and is the
latest country to join it.
• Statement 2 is correct.
Croatia joined Schengen
Zone on 1st Jan 2023. It has
also switched to the Shared
European Currency - the
EURO. Now Euro zone has
20 members (Croatia being
the 20th to join)
• 3 is correct. Croatia borders
Adriatic Sea, the
northernmost arm of the Mediterranean Sea.

Q33.
Answer: c
Explanation:

Office Complex 6, 3rd Floor, Old Rajinder Nagar New Delhi 110060|
Ph: +91-8826486658, +918826496658, | Email: students@levelupias.com
• Member States of GCC are: Saudi Arabia,
Kuwait, Bahrain, Qatar, UAE, Oman. (IRAN and
IRAQ are not members)
• A is correct: Iran is also not part of GCC, but it
is not an Arab country. Iraq is the only Arab
country bordering Persian-Gulf which is not
part of GCC.
• B is correct. All current members are
Monarchies, including three constitutional
monarchies (Qatar, Kuwait, and Bahrain).
• C is wrong. GCC was formed in 1981, after signing of Charter of Gulf Cooperation Council
OPEC was formed in 1960 to regulate the supply of Crude oil.

Q34.
Answer: a
Explanation:

• Statement 1 is correct: The IORA, formerly known as Indian Ocean Rim Initiative and
Indian Ocean Rim Association for Regional Cooperation (IOR-ARC), is an international
organization consisting of coastal states bordering Indian Ocean. It was formerly launched
in 1997. It aims to promote sustainable growth and balanced development of the region
and member states.
• Statement 2 is incorrect: Pakistan and Myanmar are not part of IORA. Member nations
(23): South Africa, Mozambique, Tanzania, Kenya, Somalia, Madagascar, Mauritius
(became member in 2014), Seychelles, Comoros, Yemen, Oman, UAE, Iran, India, Maldives
(became member in 2018), Bangladesh, Sri Lanka, Thailand, Indonesia, Malaysia,
Singapore, Australia, and France (became member in 2021). Sri Lanka is a member of
IORA, thus this statement is wrong.

Q35.
Answer: d
Explanation:
• Statement 1 is correct: Members: 19 countries and EU, Canada, US, Mexico, Brazil,
Argentina, UK, France, Germany, Italy, Turkey, South Africa, Saudi Arabia, Russia, China, India,
Japan, South Korea, Indonesia and Australia. EU is represented by European Commission and
European Central Bank.
• Statement 2 is correct: Current G20 presidency is with India. Brazil (2024) and South Africa
(2025) will have the next two Presidencies.
• Statement 3 is correct: India's Presidency Theme is "Vasudhaiva Kutumbakam" or "One Earth
One Family One Future"
• Statement 4 is incorrect: Ministry of Electronics and Information Technology (MeitY) has
launched G20 Digital Innovation Alliance (G20-DIA) to recognize and support innovators that
have digital solutions for various world economies. Recently it was introduced as a part of
India’s preparation for G20 presidency.

Office Complex 6, 3rd Floor, Old Rajinder Nagar New Delhi 110060|
Ph: +91-8826486658, +918826496658, | Email: students@levelupias.com
Q36.
Answer: a
Explanation:
• Both "ZFN" and "TALEN" are genome editing techniques used to make precise
modifications to the DNA of living organisms.
• ZFN stands for zinc finger nucleases, which are engineered proteins that can be
programmed to target and cut specific DNA sequences. TALEN stands for
transcription activator-like effector nucleases, which are also engineered proteins
that can be designed to recognize and cut specific DNA sequences.
• Both ZFN and TALEN are used in the field of genetic engineering and biotechnology
to modify the DNA of plants, animals, and other organisms for various purposes,
such as creating disease-resistant crops, developing new treatments for genetic
diseases, and studying gene function and regulation. These techniques have also
been used in clinical trials for gene therapy and hold great promise for the treatment
of a variety of genetic disorders.
Q37.
Answer: a
Explanation:
• Enriching uranium to 20% means increasing the concentration of Uranium-235 (U-235)
isotope in the uranium sample to 20% of the total uranium content. Therefore, the
correct answer is A: Preparation of nuclear fuel containing 20% Uranium-235.
• Uranium-235 is a fissile isotope that is used as fuel in nuclear reactors and nuclear
weapons. Natural uranium contains only 0.7% U-235, while the rest is mostly Uranium-
238 (U-238), which is not fissile but can be converted to fissile Plutonium-239 in a
reactor. Enrichment is the process of increasing the concentration of U-235 in uranium,
typically to about 3-5% for nuclear fuel in commercial reactors.
• Enriching uranium to 20% is a significant step because it brings the uranium closer to
weapons-grade levels, which is typically defined as 90% U-235. However, 20% enriched
uranium can also be used as fuel in some specialized types of reactors, such as research
reactors or medical isotope production reactors.

Q38.
Answer: a
Explanation:
• Diethylene glycol (DEG) and ethylene glycol (EG) are colorless and viscous liquids
which have a sweetish taste. They are popularly used in industrial products such as
paints, stationary ink, brake fluids and antifreeze.
• They are both toxic substances that can be harmful to humans if ingested or
absorbed through the skin. They are commonly used as industrial solvents and
antifreeze agents. There have been cases of mass poisoning in the Gambia, Africa.
According to US Centers for Disease Control (CDC), DEG and EG are often found as
contaminants in glycerin, which is used as a sweetener in formulations of many
pharmaceutical syrups ingested orally.

Office Complex 6, 3rd Floor, Old Rajinder Nagar New Delhi 110060|
Ph: +91-8826486658, +918826496658, | Email: students@levelupias.com
• DEG has also used illegally as a cheap substitute solvent in drug manufacturing. DEG
and EG have similar physical properties and are often used in the adulteration
process.

Q39.
Answer: b
Explanation:
Pair 1 is correctly matched: Stratospheric Observatory for Infrared Astronomy (SOFIA)
Mission: It is designed to observe the infrared universe. Flying into the stratosphere at
38,000 to 45,000 feel puts SOFIA above 99 percent of Earth’s infrared-blocking atmosphere,
allowing astronomers to study the solar system. It is the world’s largest flying telescope that
confirmed the presence of water on the Moon.
Pair 2 is correctly matched: Vikram-1 is India’s first ever privately designed and developed
rocket for commercial missions. The third stage of Vikram-1 has been named ‘Kalam-100’
after Dr. APJ Abdul Kalam. It is developed by Hyderabad based start-up Skyroot Aerospace.
Pair 3 is incorrectly matched: InSight, short for Inferior Exploration using Seismic
Investigations, Geodesy and Heat Transport, is a Mars lander. It is the first outer space
robotic explorer to study in-depth the ‘inner space’ of Mars i.e., its crust, mantle and core.
Pair 4 is incorrectly matched: NASA’s Psyche Mission will be targeted at the Psyche asteroid
that orbits the Sun in the main asteroid belt, between Mars and Jupiter.

Q40
Answer: c
Explanation:
During sunrise and sunset, the light from the sun has to travel a longer distance through the
Earth's atmosphere before it reaches our eyes. As the light travels through the atmosphere,
it gets scattered by the air molecules and other particles in the atmosphere. The scattering
of light causes the blue and green wavelengths of light to be scattered more than the longer
red wavelengths.

Therefore, during sunrise and sunset, the blue and green colors get scattered away, leaving
only the longer red wavelengths to reach our eyes. This is why the sun appears reddish in
color during these times.

Why blue and green wavelengths of light scatter more than the longer red wavelengths?
• The scattering of light is caused by interactions between light and particles in the
atmosphere, such as air molecules and dust particles. The degree of scattering
depends on the wavelength of the light.

• This is because shorter wavelengths interact more strongly with the particles,
causing them to scatter more in different directions. As a result, blue and green light
are scattered more in the atmosphere, which is why the sky often appears blue or
greenish-blue during the day.

Office Complex 6, 3rd Floor, Old Rajinder Nagar New Delhi 110060|
Ph: +91-8826486658, +918826496658, | Email: students@levelupias.com
• In contrast, the longer wavelengths of red light are scattered less in the atmosphere,
allowing more of this light to travel straight to our eyes during sunrise and sunset.

Q41.
Answer: d
Explanation:
Synthetic biology is an interdisciplinary field of biology and engineering that involves the
design, construction, and manipulation of biological systems that do not exist in nature. It
aims to create new biological parts, devices, and systems using engineering principles, and
to apply these systems to solve real-world problems. Synthetic biology combines molecular
biology, biotechnology, genetics, and computer science, among other fields, to create new
organisms or modify existing ones for specific purposes.
It has a wide range of applications across many different fields, including medicine,
biotechnology, agriculture, energy, and environmental science. Here are a few examples of
its current applications:
• Creating new medicines: Synthetic biology is being used to create new drugs and therapies
for a variety of diseases, including cancer, Alzheimer's, and HIV.
• Developing new materials: Synthetic biology is being used to develop new materials for use
in everything from clothing to construction to electronics.
• Producing biofuels: Synthetic biology is being used to engineer microorganisms that can
produce biofuels like ethanol and butanol from renewable sources like biomass and algae.
• Enhancing food production: Synthetic biology is being used to create new strains of crops
that are more resistant to pests and disease, have higher yields, and can survive in adverse
environmental conditions.
• Environmental cleanup: Synthetic biology is being used to develop bacteria and other
organisms that can break down pollutants and other harmful substances in the
environment, such as oil spills.

Overall, the potential applications of synthetic biology are vast and varied, and the field is
constantly expanding as researchers explore new possibilities.

Q42.
Answer: a
Explanation:
• The westerlies in the Southern Hemisphere are generally stronger and more persistent than
in the Northern Hemisphere. This is because the Southern Hemisphere has fewer
landmasses and more unobstructed ocean, allowing the westerlies to blow more freely and
consistently. In addition, the Antarctic Circumpolar Current, which flows continuously
around Antarctica in a westerly direction, helps to maintain the strength and persistence of
the westerlies in the Southern Hemisphere. So, statement 1 is correct.
• No, the trade winds are not responsible for the winter rains on the Mediterranean coast.
The trade winds blow from east to west in the tropics and subtropics, and they are strongest
and most consistent over the ocean. However, the Mediterranean Sea is not directly
affected by the trade winds. The westerlies are responsible for winter rain on the
Mediterranean coast. So, statement 2 is not correct.
• The winter rains on the Mediterranean coast are primarily caused by the region's
Mediterranean climate, which is characterized by mild, wet winters and hot, dry summers.

Office Complex 6, 3rd Floor, Old Rajinder Nagar New Delhi 110060|
Ph: +91-8826486658, +918826496658, | Email: students@levelupias.com
During the winter months, the mid-latitude cyclones can bring rain and storms to the
Mediterranean region, which contributes to the winter rainfall.

Q43.
Answer: a
Explanation:
• Statement 1 is incorrect because the atmosphere would not come to an immediate
standstill. Instead, the sudden change in atmospheric circulation caused by the Earth's
stoppage would likely result in severe weather events such as hurricanes and tornadoes.
• If the Earth stopped spinning immediately, objects on the Earth's surface would indeed be
thrown away with great speed due to the inertia of their motion. So, statement 2 is correct.
• Every spot-on Earth would have permanent daytime or nighttime, depending on which side
of the Earth was facing the sun at the time of the stoppage. If the Earth stopped spinning
immediately, every spot-on Earth would indeed have permanent daytime or nighttime all
year long. So, statement 3 is correct.
• Statement 4 is incorrect accurate because the sudden stoppage of the Earth's rotation
would not directly cause a rapid drop in temperature. However, the temperature changes
resulting from the permanent daytime or nighttime could lead to significant temperature
variations across the planet.

Q44.
Answer: a
Explanation:
• Temperature over oceans decreases from the equator to the poles due to the decrease in
the amount of insolation received towards the poles. So, Statement 1 is correct.
• Oceans in the southern hemisphere receive less heat due to their greater distance from the
equator and a relatively small landmass in the Southern Hemisphere compared to the
Northern Hemisphere. This allows for greater mixing of ocean currents, which helps to
distribute heat more evenly throughout the ocean. So, Statement 2 is correct.
• Winds blowing from land to sea can cause the sea surface temperature to decrease rather
than increase because of the upwelling of cold water. This is because the winds push the
warmer surface water away from the shore, allowing the cooler water from below to rise to
the surface. So, Statement 3 is incorrect.

Q45.
Answer: b
Explanation:
• The Konkan Coast and Malabar Coast of India receive rainfall primarily from the South-West
monsoon, but not from the winter monsoon (also known as the North-East monsoon). The
Coromandel Coast receives rainfall primarily from the North-East monsoon, but not from
the South-West monsoon.
• The coastal region of Karnataka, also known as the Karavali coast, is the only coastal region
in India that receives significant rainfall from both the South-West monsoon and North-East
monsoon. North-East monsoon brings rain from October to December. Northeastern
monsoon brings substantial rainfall to south-inner Karnataka cities such as Mangalore,
Udupi, and Karwar. Therefore, the correct answer is (B) Karnataka Coast.

Office Complex 6, 3rd Floor, Old Rajinder Nagar New Delhi 110060|
Ph: +91-8826486658, +918826496658, | Email: students@levelupias.com
Q46.
Answer: c
• The vegetation type described in the passage is that of moist deciduous forests. These
forests are characterized by the shedding of leaves during the dry season to conserve water.
The main species found in these forests are Teak, Sal, Laurel, Rosewood, and Bamboo.
These forests are found in the northeastern states along the foothills of Himalayas, eastern
slopes of the Western Ghats and Odisha.
• The region that does not have the above-mentioned vegetation is (C) Eastern Rajasthan.
Eastern Rajasthan is predominantly covered with thorn forests and scrublands.

Q47.
Answer: b
Explanation:
• The Indus, the Sutlej, the Kosi, and the Brahmaputra are antecedent to all the three ranges
of the Himalayas as they cut across the Greater, the Lesser and the Outer Himalayan ranges.
It is believed that these rivers were present before the folding and upliftment of the
Himalayan ranges.
• Beas, Ravi, Ganga, and Ghaghara are not older than the Himalayas because these rivers
originate from the Himalayas. So, option (B) is correct.

Q48.
Answer: a
Explanation:
Under Carbon Border Adjustment Mechanism, European Union has proposed a new tax
namely, Carbon Border Tax.
Carbon Border Tax (proposal by EU in 2021):
1. It is defined as a penalty tax to discourage import of carbon-intensive goods such as steel,
aluminium, cement, fertilizers and electricity via carbon border adjustment mechanism.
2. The aim is to help slash the EU’s overall greenhouse gas emissions 55% below 1990 levels
by 2030.
3. The EU Commission is pushing for the world’s first border tax on imported goods. It plans
to levy the tax in a phased manner from 2026.

Q49.
Answer: d
Explanation:
Kunming Montreal Global Biodiversity Framework (GBF) (2022): Some features:
1. The GBF includes four goals and 23 targets to address the loss of biodiversity and restore
natural ecosystems by 2030.
2. Targets of GBF:
(a) Protecting at least 30% of the world’s land, water, coastal, and marine areas.
(b) Restoring at least 30% of degraded terrestrial, inland water, coastal, and marine
ecosystems.
(c) Reducing or eliminating the loss of areas with high biodiversity importance.
(d) Cutting global food waste in half and reducing overconsumption and waste.

Office Complex 6, 3rd Floor, Old Rajinder Nagar New Delhi 110060|
Ph: +91-8826486658, +918826496658, | Email: students@levelupias.com
(e) Reducing the use of pesticides and hazardous chemicals by half.
(f) Phasing out or reforming subsidies that harm biodiversity by at least USD500 billion per
year and scaling up incentives for conservation and sustainable use.
(g) Mobilizing at least USD200 billion per year in funding from all sources for biodiversity-
related causes.
(h) Increasing international financial flows to developing countries to at least USD20 billion
per year by 2025 and at least USD30 billion per year by 2030.
(i) Preventing the introduction of invasive species and reducing the establishment of other
known or potential invasive species.
3. Goals of GBF:
(a) Maintaining, enhancing, or restoring the integrity, connectivity, and resilience of all
ecosystems.
(b) Halting human-induced extinction of threatened species and reducing the extinction rate
and risk of all species.
(c) Maintaining genetic diversity within populations of wild and domesticated species.
(d) Sustainably using and managing biodiversity and valuing nature’s contributions to
people.

Q50.
Answer: d
Explanation:
Supreme Court directed chief secretaries of two states viz. Gujarat and Rajasthan:
Installation of bird diverters (reflectors like structures stung on power cables) in priority
areas.
Great Indian Bustards:
1. Great Indian Bustards (GIBs) are considered as flagship/keystone grassland species and
State’s bird of Rajasthan. (Statement 1 is correct)
2. Their population are confined mostly in Rajasthan and Gujarat. Small populations occur in
Maharashtra, Karnataka and Andhra Pradesh.
3. The birds are under threat due to collision/electrocution with power transmission lines,
hunting (still prevalent in Pakistan), habitat loss and alteration as a result of widespread
agricultural expansion etc.
4. IUCN Status: Critically Endangered; Appendix-I of CITES; and Appendix-I of Wildlife
Protection Act, 1972. (Statement 2 is correct)
5. Appendix-I of CMS (Convention of Migratory Species).
6. Distribution: Its population of about 150 in Rajasthan accounts for 95% of its total world
population. They are mainly in Jaisalmer district of Rajasthan, including the Desert National
Park which is the natural habitat of the species. (Statement 3 is correct).

Q51.
Answer: d
Explanation:
Statement 1 is correct: The Great Barrier Reef is located in the North-Eastern coast of
Australia and is known to be the world's largest structure made up of living organisms. It
consists of thousands of individual reefs and islands that stretch for over 2,300 kilometers
along the Queensland coast.

Office Complex 6, 3rd Floor, Old Rajinder Nagar New Delhi 110060|
Ph: +91-8826486658, +918826496658, | Email: students@levelupias.com
Statement 2 is correct: It is included in the list of World Heritage Sites by UNESCO. This
recognition was given to the reef in 1981, and it is based on its exceptional natural beauty
and ecological significance. The Great Barrier Reef is home to over 1,500 species of fish, 600
types of coral, and many other marine species.
Statement 3 is correct: In 2022, Corals have turned white across all four of the reef’s main
areas, despite the cooling influence of the La Niña climate phenomenon, in the natural
wonder's sixth mass bleaching event of modern times.
Mass bleaching events occur when ocean temperatures rise above normal levels, causing
the coral to expel the algae that live inside their tissues, leading to a loss of coluor and
potentially even death. The Great Barrier Reef experienced mass bleaching events in 2016
and 2017, which were among the most severe on record. In 2016, around two-thirds of the
coral in the northern part of the reef died due to bleaching, and in 2017, the central part of
the reef also suffered severe bleaching. These events were caused by a combination of high
sea temperatures and other environmental stresses.

Q52.
Answer: a
Explanation:
The Ministry of Environment, Forest and Climate Change (MoEFCC) launched Mission
Sahbhagita in 2022 with the aim of ensuring the effective management of 75 wetlands of
national and international significance through a participatory approach.

Under Mission Sahbhagita, the Ministry plans to involve local communities, stakeholders,
and other relevant organizations in the conservation and management of wetlands, with the
goal of promoting a society ownership approach. The mission also aims to build capacity and
create awareness among stakeholders on the importance of wetlands, their conservation,
and sustainable use.

The Ramsar Convention is an international treaty signed in 1971, aimed at the conservation
and wise use of wetlands. The Convention promotes a participatory approach to wetland
management, involving local communities, stakeholders, and indigenous peoples in the
conservation and management of wetlands.

Q53.
Answer: c
Explanation:
Statement 1 is incorrect: Reserve Bank broadly defines CBDC as the legal tender issued by a
central bank in a digital form. It is akin to sovereign paper currency but takes a different
form, exchangeable at par with the existing currency and shall be accepted as a medium of
payment, legal tender and a safe store of value. CBDCs would appear as liability on a central
bank’s balance sheet.
Statement 2 is correct: CBDC can be structured as ‘token-based’ or ‘account-based’. A
token-based CBDC is a bearer-instrument like banknotes, meaning whosoever holds the
tokens at a given point in time would be presumed to own them. In contrast, an account-
based system would require maintenance of record of balances and transactions of all
holders of the CBDC and indicate the ownership of the monetary balances.

Office Complex 6, 3rd Floor, Old Rajinder Nagar New Delhi 110060|
Ph: +91-8826486658, +918826496658, | Email: students@levelupias.com
Statement 3 is correct: CBDC can be classified into two broad types viz. general purpose or
retail (CBDC-R) and wholesale (CBDC-W). Retail CBDC would be potentially available for use
by all viz. private sector, non-financial consumers and businesses while wholesale CBDC is
designed for restricted access to select financial institutions.

Q54.
Answer: c
Explanation:
Both statements are correct:
Green bonds are bonds issued by any sovereign entity, inter-governmental groups or
alliances and corporates with the aim that the proceeds of the bonds are utilised for
projects classified as environmentally sustainable.
• The issuance of Sovereign Green Bonds will help Government of India (GoI) in tapping the
requisite finance from potential investors for deployment in public sector projects aimed at
reducing the carbon intensity of the economy.
• Payments of principal and interest on the issuances under this Framework are not
conditional on the performance of the eligible projects. Investors in bonds issued under this
Framework do not bear any project related risks.

Q55.
Answer: a
Explanation:
Ministry of Housing & Urban Affairs launched a scheme PM Street Vendor's AtmaNirbhar
Nidhi (PM SVANidhi) to empower Street Vendors by not only extending loans to them, but
also for their holistic development and economic upliftment. The scheme intends to
facilitate collateral free working capital loans of up to INR10,000/- of one-year tenure, to
approximately 50 lakh street vendors, to help resume their businesses in the urban areas,
including surrounding peri-urban/rural areas.

Q56.
Answer: d
Explanation:
The prices of petrol can vary from state to state due to a number of factors, including
transportation costs, state-specific taxes, and differences in the cost of distribution. One of
the significant factors that contribute to the difference in fuel prices across different states
is the state-specific tax, i.e., VAT, which varies from state to state.

Each state government imposes its own VAT on petrol, which is a percentage of the price of
the fuel. The VAT rates can vary from state to state and range from 20% to 39%. For
instance, in Delhi, the VAT on petrol is currently fixed at 30%, while in Maharashtra, it is
fixed at 26%. The difference in the VAT rate can cause a difference in the retail price of
petrol in different states.

Office Complex 6, 3rd Floor, Old Rajinder Nagar New Delhi 110060|
Ph: +91-8826486658, +918826496658, | Email: students@levelupias.com
Q57.
Answer: d
Explanation:
Statement 1 is correct: One of the major benefits of improved digital infrastructure is
increased financial inclusivity. With digital infrastructure in place, individuals and
businesses in remote or underserved areas can gain access to banking and financial services.
This can help to bring people into the formal financial system, making it easier for them to
save money, access credit, and engage in other financial activities that can improve their
economic prospects.
Statement 2 is correct: Improved digital infrastructure can also help to reduce tax evasion.
Digital payment systems can be used to track financial transactions more effectively, making
it more difficult for people and businesses to engage in undeclared or illegal activities.
Additionally, digital tax filing systems can make it easier for individuals and businesses to file
their taxes and reduce errors, making the tax collection process more efficient.
Statement 3 is correct: Another key outcome of improved economic digital infrastructure is
increased formalization of the economy. With digital systems in place for payments, taxes,
and other financial transactions, it becomes easier for businesses to operate in a formal,
transparent manner. This can help to reduce informal economic activity and bring more
businesses into the formal economy.
Statement 4 is correct: Finally, improved economic digital infrastructure can help to
improve financial and digital literacy. As people gain access to digital financial systems,
they are forced to learn how to use them effectively. This can improve financial literacy and
help people to make better financial decisions. Similarly, as people engage more with digital
technologies, they become more comfortable with using them, which can improve digital
literacy and help people to access a wider range of digital services.

Q58.
Answer: b
Explanation:
Pent-up demand refers to the phenomenon where consumer demand for a product or
service accumulates over time due to factors such as supply shortages, economic
uncertainty, or other market conditions that discourage immediate purchasing. When these
conditions change, the demand is released and results in a sudden surge of buying activity.
This often leads to a temporary increase in sales and can create a backlog of orders or
backlogs in the supply chain. Pent-up demand can occur in various industries and can be a
sign of a recovering economy or a shift in consumer behavior.
1. Demonetization can lead to a temporary shortage of cash in the economy, which can
discourage immediate purchasing. (1 is correct)
2. Pandemic induced lockdowns can cause economic uncertainty as it may lead to closure
of some business activities. (2 is correct)
3. Increased government spending can lead to increased demand for goods and services,
which in turn can lead to increased production, employment, and economic growth, hence
its not a precursor to pent-up demand. Additionally, increased government spending can
also lead to increased investment from private businesses, as they are more likely to invest
in an economy that is experiencing growth. (3 is incorrect)

Office Complex 6, 3rd Floor, Old Rajinder Nagar New Delhi 110060|
Ph: +91-8826486658, +918826496658, | Email: students@levelupias.com
4. Restrictions in global supply chains can lead to supply shortages which may further cause
high prices of some commodities and thus discourage consumers. (4 is correct)

Q59.
Answer: c
Explanation:
Government of India Act 1919 was based on what are popularly known as the Montague-
Chelmsford Reforms. In August 1917, the British government for the first time declared that
its objective was to gradually introduce responsible government in India, but as an integral
part of the British Empire.
Statement 1 is correct: Under the 1919 Act, the Indian Legislative Council at the Centre was
replaced by a bicameral system consisting of a Council of State (Upper House) and a
Legislative Assembly (Lower House). Each house was to have a majority of members who were
directly elected. So, direct election was introduced, though the franchise was much restricted
being based on qualifications of property, tax or education.
In the earlier Act of 1909, an element of election was present, but the elected members were
elected indirectly. The local bodies elected an electoral college who would elect members of
the provincial legislative councils. These members would, in turn, elect the members of the
Central legislative council.
Government of India Act 1935, with 451 clauses and 15 schedules, contemplated the
establishment of an All-India Federation in which Governors’ Provinces and the Chief
Commissioners’ Provinces and those Indian states which might accede to be united were to
be included.
Some important features of the act were:
• Dyarchy in the provinces was abolished and provinces were given autonomy, i.e., the
distinction between Reserved and Transferred Subjects was abolished and full
responsible government was established, subject to certain safeguards.
• Provinces derived their power and authority directly from the British Crown. They
were given independent financial powers and resources. Provincial governments
could borrow money on their own security. (Statement 2 is correct)
• Provincial legislatures were further expanded. Bicameral legislatures were provided in
the six provinces of Madras, Bombay, Bengal, United Provinces, Bihar and Assam, with
other five provinces retaining unicameral legislatures.
• The principles of ‘communal electorates’ and ‘weightage’ were further extended to
depressed classes, women and labour.

Q60.
Answer: d
Explanation:
The above statements describe Subhash Chandra Bose.
Subhash Chandra Bose talked of economic development of the country through planning
and was instrumental in setting up a National Planning Committee later.
The National Planning Committee was formed by Subhas Chandra Bose and Jawaharlal
Nehru in 1938 as a response to the need for economic planning in India. The committee was
a group of Indian leaders and experts from various fields, including economists, scientists,

Office Complex 6, 3rd Floor, Old Rajinder Nagar New Delhi 110060|
Ph: +91-8826486658, +918826496658, | Email: students@levelupias.com
and social workers, who were tasked with developing a comprehensive plan for India's
economic development. In 1942, Gandhi called Bose the “Prince among the Patriots”.
He wrote ‘An Indian Pilgrim’. It was written towards the end of 1937 during his Europe trip,
after being nominated the President of the Indian National Congress, An Indian Pilgrim
traces Bose's life story from birth till his resignation from the Indian Civil Service. The other
works of S C Bose are: Dreams of a Youth, ON to Delhi, A Beacon across Asia, Ideas of Nation,
The Indian Struggle etc.

Q61.
Answer: c
Explanation:
Statement 1 is correct: After the Nagpur session of Congress in 1920, the provincial congress
committees were constituted on the basis of language. This was a significant step towards
recognizing the linguistic diversity of India and giving representation to linguistic groups in the
Indian National Congress.
In 1920, Mahatma Gandhi made significant changes to the Congress organization, which
played a crucial role in India's struggle for independence. Here are some of the changes that
were made:
• Non-Cooperation Movement: Gandhi launched the Non-Cooperation Movement,
which called for Indians to boycott British goods, educational institutions, and law
courts.
• Swaraj: The Congress adopted the goal of swaraj, which means self-rule or
independence, as its ultimate objective.
• Membership: The Congress opened its membership to anyone who was willing to pay
a nominal fee and subscribe to the Congress's aims and objectives.
• Khadi and Charkha: Gandhi emphasized the importance of khadi, or hand-spun and
hand-woven cloth, as a symbol of Indian self-reliance and economic independence.
He also encouraged the use of the charkha, or spinning wheel, as a means of self-
sufficiency.
• Satyagraha: Gandhi introduced the concept of satyagraha, or non-violent resistance,
as a means of opposing British rule.
• Working Committee: The Congress established a Working Committee, which was
responsible for the day-to-day functioning of the organization.
• Congress Sessions: The Congress held annual sessions, which provided a platform for
leaders from across India to discuss and debate important issues related to the
freedom struggle.
These changes helped to transform the Congress into a mass movement that could mobilize
millions of Indians in the struggle for independence.

Statement 2 is correct. In 1948, the demand for the formation of provinces on the basis of
language was rejected by the Congress. This decision was made by the Congress Working
Committee and was based on the argument that the formation of linguistic provinces could
lead to the fragmentation of India on linguistic lines. However, the demand for linguistic
states continued to gain momentum, and eventually, the first linguistic state of Andhra
Pradesh was created in 1953.

Office Complex 6, 3rd Floor, Old Rajinder Nagar New Delhi 110060|
Ph: +91-8826486658, +918826496658, | Email: students@levelupias.com
JVP committee was set up by the Congress to study the recommendations of Linguistic
Provinces Commission (Dar Commission). It comprised Jawaharlal Nehru, Vallabhbhai Patel
and Pattabhi Sitaramayya. The committee gave its report on 1 April 1949 and said that time
was NOT suitable for formation of new provinces.The committee rejected the linguistic factor
of reorganization of the states. The Committee reaffirmed the Dhar Commission's position in
its report in 1949. The committee recommended the reorganization of States on the basis of
security, unity, and economic prosperity of the nation.
Fazl Ali commission also known as the States Reorganization Commission was formed in 1953
to ponder upon the demands of different states. The states just after independence
demanded reorganization of the whole country into different parts based on the language.
Preservation and strengthening of the unity and security of the country, linguistic and cultural
homogeneity were the recommendations given by the commission in any scheme of
reorganization of states.

Q62.
Answer: b
Explanation:
Vaikom Satyagraha was a temple entry movement of depressed classes organised in Kerala in
1924-25.
Statement 1 is incorrect: Madhavan, K.P. Kesava Menon who was the then secretary of Kerala
Pradesh Congress Committee and Congress leader and educationist K. Kelappan (also known
as Kerala Gandhi) are considered the pioneers of the Vaikom Satyagraha movement.
Subramania Bharathi (1882-1921) was a Tamil writer, poet, journalist, Indian independence
activist, social reformer and polyglot. He was bestowed the title "Bharathi" for his excellence
in poetry. He was a pioneer of modern Tamil poetry and is considered one of the greatest
Tamil literary figures of all time. He is popularly known by his mononymous title "Bharathi/
Bharathiyaar," and also by the other title "Mahakavi Bharathi" ("the great poet Bharathi").
His numerous works included fiery songs kindling patriotism during the Indian Independence
movement. He fought for the emancipation of women, against child marriage, vehemently
opposed the caste system, and stood for reforming society and religion. He was also in
solidarity with Dalits and Muslims.

Sir PittiTheagaraya Chetty KCSI (27 April 1852 – 28 April 1925) was an Indian lawyer,
industrialist and a prominent political leader from the erstwhile Madras province. He was one
of the founders of the Justice Party in 1916 along with C. NatesaMudaliar, Dr. T. M. Nair.

Statement 2 is correct: Lower castes like the Ezhavas and Pulayas were considered polluting
and various rules were in place to distance them from upper castes.Most of the great temples
in the princely state of Travancore had for years forbidden lower castes (untouchables) not
just from entering, but also from walking on the surrounding roads. The agitation was
conceived by the Ezhava Congress leader and a follower of Sri Narayana Guru, T. K. Madhavan.
It demanded the right of the Ezhavas and 'untouchables' to use roads around the Vaikom
Temple.

Office Complex 6, 3rd Floor, Old Rajinder Nagar New Delhi 110060|
Ph: +91-8826486658, +918826496658, | Email: students@levelupias.com
Statement 3 is correct: Mahatma Gandhi arrived at Vaikom in March 1925, held a series of
discussions with leaders of various caste groups and met Maharani Regent at her Varkala
camp.
The Vaikom Satyagraha was officially withdrawn on November 30, 1925 after consultations
between Gandhi and W.H. Pitt (police commissioner of Travancore).
A compromise was reached following the release of all prisoners and grant of access to roads.
Finally as a result of this satyagraha, in 1936, the historic Temple Entry Proclamation was
signed by the Maharaja of Travancore which removed the age-old ban on the entry of
temples.

Q63.
Answer: a
Explanation:
The Indian League was started in 1875 by Sisir Kumar Ghosh with the object of “stimulating
the sense of nationalism amongst the people” and of encouraging political education. Hence,
pair 1 is not correctly matched.
The Poona Sarvajanik Sabha was founded in 1867 by Mahadeo Govind Ranade and others,
with the object of serving as a bridge between the government and the people. Hence, Pair
2 is correctly matched.
The BangabhashaPrakasika Sabha was formed in 1836 by Gourishankar Tarkabagish,
associates of Raja Ram Mohan Roy in 1836 in Calcutta, not by Raja Rammohan Roy himself.
Hence, pair 3 is not correctly matched.
British Indian Association was established on 29 October 1851 in Kolkata, India with
Radhakanta Deb as its first President. The first general secretary of the association was
Debendranath Tagore. The association was exclusively composed of Indians and it worked
towards increasing the welfare of Indians. Its members included Kristo Das Pal, Peary Chand
Mitra, and Ramgopal Ghosh. Hence, Pair 4 is incorrectly matched.

Q64.
Answer: b
Explanation:
Pair 1 is correctly matched: DALLE 2.0 or "Diverse Aligned Logical Linear Entities" is a large-
scale neural network language model developed by OpenAI. It is an extension of the original
DALL-E, which was capable of generating unique images from textual descriptions, by
incorporating natural language processing (NLP) capabilities to better understand and
reason about the relationships between objects and concepts in text. DALLE 2.0 can
generate images from textual prompts, and can also complete textual prompts or generate
new text in a variety of styles and contexts.
Pair 2 is correctly matched: Dubbed as an open-source and distributed or federated social
network, Mastodon allows users to create their own networks or ‘instances’, essentially
handing control and ownership of data back to the user. Each instance has its own policies
related to content moderation, code of conduct, terms of service and privacy etc.
Pair 3 is incorrectly matched: ELISA stands for Enzyme-Linked Immunosorbent Assay. It is a
laboratory technique used to detect and measure the presence of specific proteins,
antibodies, or antigens in a sample. ELISA is a commonly used diagnostic tool in fields such
as immunology, microbiology, and medicine. It belongs to the field of Immunology

Office Complex 6, 3rd Floor, Old Rajinder Nagar New Delhi 110060|
Ph: +91-8826486658, +918826496658, | Email: students@levelupias.com
Q65.
Answer: d
Explanation:
Statement 1 is correct: Biofortification is a process in which the nutritional quality of crops
is improved through conventional plant breeding or genetic engineering techniques. Gene
technology can be used to introduce genes responsible for the production of specific
nutrients in crops.
Statement 2 is correct: The extrusion method used for rice fortification involves only
physical change. During the extrusion process, rice grains are subjected to high temperature
and pressure, which causes the starch in the rice to gelatinize. Vitamins and minerals can
then be added to the gelatinized rice, which is then dried and milled.
Statement 3 is correct: Currently, no genetically modified biofortified crop is used in India to
ensure nutritional security. However, several conventional biofortified crops, such as iron-
rich pearl millet and zinc-rich wheat, are being promoted in India to address micronutrient
deficiencies in the population.

Q66.
Answer: d
Explanation:
All Statements are correct:
• Free Space Optical Communications (FSOC) is the wireless transmission of data via a
modulated optical beam directed through free space, without fiber optics or other
optical systems guiding the light. FSOC links use beams of light to deliver high-speed,
high-capacity connectivity over long distances.
• WiMAX, the Worldwide Interoperability for Microwave Access, is a standard based
technology aimed at providing wireless data over long distances in a variety of ways,
from point-to-point links to full mobile cellular type access.WiMAX can provide two
types of wireless service. They are non-line-of-sight and Line-of-sight.
• LiFi, Light Fidelity, is a visible light communications system transmitting wireless
internet communications at very high speeds. Unlike Wi-Fi, which uses radio waves,
Li-Fi runs on visible light. The technology makes a LED light bulb emit pulses of light
that are undetectable to the human eye and within those emitted pulses, data can
travel to and from receivers. Then, the receivers collect information and interpret
the transmitted data.

Q67.
Answer: c
Explanation:
INS Mormugao:
1. Named after the historic port city of Goa on the West coast, Mormugao coincidentally
when Goa celebrated 60 years of liberation from the Portuguese rule.
2. Features:
(a) Packed with sophisticated state-of-the-art weapons and sensors such as Surface-to-
Surface Missile and Surface-to-Air Missiles. (Statement 1 is correct)

Office Complex 6, 3rd Floor, Old Rajinder Nagar New Delhi 110060|
Ph: +91-8826486658, +918826496658, | Email: students@levelupias.com
(b) Fitted with a modern Surveillance Radar which provides target data to the gunnery
weapon systems.
(c) Its Anti-Submarine Warfare capabilities viz. indigenously-developed Rocket Launchers,
Torpedo Launchers and the ASW helicopters.
(d) Equipped to fight under Nuclear, Biological and Chemical warfare conditions.
(Statement 2 is correct)

Q68.
Answer: a
Explanation:
Statement 1 is incorrect: It is a live attenuated strain derived from an isolate of
Mycobacterium bovis and has been used widely across the world as a vaccine for
tuberculosis. Currently, it is the only licensed vaccine available for the prevention of TB. It is
the world's most widely used vaccine with about 120 million doses every year.
Statement 2 is incorrect: Generally, the farther a country is from equator, the higher is the
efficacy. Therefore, it has high efficacy in UK, Norway, Sweden, and Denmark; and little or
no efficacy in countries on or near the equator like India, Kenya, and Malawi, where the
burden of TB is higher. These regions also have higher prevalence of environmental
mycobacteria.
Statement 3 is correct: UIP provides vaccination against 12 Vaccine-Preventable Diseases
(VPD) i.e. diphtheria, Whooping cough, tetanus, polio, tuberculosis(BCG), hepatitis B,
meningitis and pneumonia, Haemophilus influenzae type B infections, Japanese encephalitis
(JE), rotavirus vaccine, pneumococcal conjugate vaccine (PCV) and measles-rubella (MR).

Q69.
Answer: b
Explanation:
• The existence of perennial high pressure over the tropical west margins of the continents is
the primary reason for the occurrence of deserts in these regions. This high pressure results
in stable atmospheric conditions, which inhibit the formation of clouds and precipitation,
leading to arid or semi-arid conditions. So, Statement 1 is correct.
• Cold ocean currents cool the surrounding air and reduce atmospheric instability. This can
make it more difficult for clouds to form and precipitation to occur. So, Statement 2 is
correct.
• It is Off-shore winds blow over this region. This wind carries no moisture thus causes no
precipitation. The desert biome is an ecosystem that forms due to the low level of rainfall it
receives each year. So, Statement 3 is not correct.
• Rain shadow effect: Mountains located near these regions can block moisture from reaching
the area, creating a rain shadow effect that results in dry conditions.

Q70.
Answer: b
Explanation:
• Sugarcane requires well-drained fertile soil for optimum growth, but abundant rainfall are
not essential as it can damage the crop through waterlogging, soil erosion, disease and pest
outbreak.

Office Complex 6, 3rd Floor, Old Rajinder Nagar New Delhi 110060|
Ph: +91-8826486658, +918826496658, | Email: students@levelupias.com
• Regular irrigation is necessary to ensure adequate soil moisture for growth. Flood irrigation,
where fields are flooded with water, is the most common method of irrigation for rice
cultivation in India.
• High humidity and low temperatures are not favorable for sugarcane cultivation because
they create conditions that are conducive to the growth of fungal diseases that can affect
the growth and yield of sugarcane crops.
• The sugar industry is the second largest agro-based industry in India after cotton.
• India is the second largest producer of sugarcane after Brazil.
• Top Sugarcane Producing States: Uttar Pradesh, Maharashtra, Karnataka, Tamil Nadu, Bihar.

Q71.
Answer: a
Explanation:
Tropical evergreen forests in India are found in areas with high rainfall, typically in excess of
200 cm per year. These forests are dominated by tall trees with broad, dark green leaves
and dense canopies, making them one of the most biodiverse ecosystems in the world.
These forests are home to a diverse range of flora and fauna, including species that are
endemic to India. Tropical evergreen forests are generally found in the Western Ghats, the
Andaman and Nicobar Islands, and parts of northeastern India. Therefore, option D, which
includes all four statements, is the correct answer.

Q72.
Answer: d
Explanation:
1. Shevroy Hills (T.N ):The Shevroy hills, are a towering mountain range near the city of
Salem, in Tamil Nadu state, southern India. It is one of the major hill stations in the Eastern
Ghats.
2. Javadi Hills (T.N): The Javadi hills are an extension of the Eastern Ghats in the state of
Tamil Nadu in southeastern India.
3. Nallamalla Hills (Andhra Pradesh): The Nallamalas are a section of the Eastern Ghats
which forms the eastern boundary of Rayalaseema region of the state of Andhra Pradesh
and Nagarkurnool district of the state of Telangana, in India.
4. Garhjat Hills (Odisha): The Garhjat Hills is a mountain range formed by a series low-lying
hills, plateaux, ridges and meadows that stretch into Odisha from the Utkal Plains in the
Chotanagpur region of Jharkhand and the Chhattisgarh Plains.

Q73.
Answer: c
Explanation:
• Lithium (Li), sometimes also referred to as ‘White gold’ due to its high demand for
rechargeable batteries, is a soft and silvery-white metal.
• Lithium is the lightest known metal. It has a density of 0.534 g/cm3 (half as dense as water)
and has the lowest melting points of all metals and a high boiling point. Anode, cathode,
electrolyte and separator are the main components of a lithium-ion (rechargeable) battery.
So, statement 1 is correct.

Office Complex 6, 3rd Floor, Old Rajinder Nagar New Delhi 110060|
Ph: +91-8826486658, +918826496658, | Email: students@levelupias.com
• Lithium-ion cells is that their rate of self-discharge is much lower than that of other
rechargeable cells such as Ni-Cad (nickel– cadmium) etc. It is typically around 5% in the first
4 hours after being charged but then falls to a figure of around 1 or 2% per month. So,
statement 2 is correct.
• Major Global Lithium Reserves: Chile > Australia > Argentina are top countries with Li
reserves.
• Lithium Triangle: Chile, Argentina, Bolivia. So, statement 3 is not correct.
• Recently, Geological Survey of India has for the first-time established Lithium inferred
resources (G3) of 5.9 million tonnes in Salal-Haimana area of Reasi District of Jammu &
Kashmir (UT). So, statement 4 is not correct.

Q74.
Answer: c
Explanation:
· Space debris, also called space junk, is artificial material that is orbiting Earth but is no
longer functional. This material can be as large as a discarded rocket stage or as small as a
microscopic chip of paint.
· Much of the debris is in low Earth orbit, within 2,000 km of Earth’s surface, though some
debris can be found in geostationary orbit 35,786 km above the Equator.
· The free-floating space debris is a potential hazard for operational satellites and colliding
with them can leave the satellites dysfunctional. This is referred to as Kessler Syndrome,
named after NASA scientist Donald Kessler in 1978.
Clearspace-1 (of European Space Agency), which is scheduled to launch in 2025, will be the
first space mission to eliminate debris from orbit. Hence, option C is correct.

Q75.
Answer: d
Explanation:

• The Asian Infrastructure Investment Bank


(AIIB) is a multilateral development bank
with a mission to improve social and
economic outcomes in Asia. It aims to
support the building of infrastructure in
the Asia Pacific Region.
• Members and Founding Members: The
institution currently has more than 100
members (Regional members: 46, non-
regional members: 39, Prospective
Members: 18).
• All of Europe, Canada, Australia, New Zealand are also members.
• Major economies that are not members of AIIB are USA, Japan, Mexico, and Nigeria
(Nigeria has plans to join AIIB), 4 is incorrect.
• Reasons for Formation - Slow pace of reforms and governance in global financial
institutions such as IMF, World Bank, and Asian Development Banks

Office Complex 6, 3rd Floor, Old Rajinder Nagar New Delhi 110060|
Ph: +91-8826486658, +918826496658, | Email: students@levelupias.com
• Biggest Beneficiary: India has been the biggest beneficiary of the bank. Out of $20 billion in
loans issued by the Beijing-based Asian Infrastructure Investment Bank (AIIB), about $6
billion has gone to India.
• AIIB is supporting many projects including the Chennai Metro Corridors Four and Five;
Chennai Peripheral Ring Road; the Delhi-Meerut Regional Rapid Transit System etc.
• The Contingent Reserve Arrangement (CRA) is a financial safety net established by the BRICS
countries (Brazil, Russia, India, China, and South Africa) in 2015. The CRA is meant to provide
a pool of financial resources that can be used by BRICS countries to help each other in the
event of balance of payments difficulties or other financial emergencies. So, statement 3 is
wrong.

Q76.
Answer: a
Explanation:

• India has jumped six places and ranked 38 out of 139 countries in the 7th edition of World
Bank’s Logistics Performance Index 2023.
• It is one of the important Index published by the World Bank. In 2018 index, India's rank
was 44th and in 2014 it was 54th.
• Singapore (1) and Finland (2) are the best performing countries. China is not even in top
10, it is ranked 19th.

Q77.
Answers: c
Explanation:
• Location: Between Russia (Asia) and Alaska (North America); Only marine gateway between
icy Arctic and the Pacific Ocean. It joins the Arctic Ocean and Bering Strait.
• At its narrowest point the strait is only 55 miles wide.
• Biodiversity: Though the strait is narrow, it is teaming with wildlife - Beluga whales,
bowhead whales, walruses, polar bears, ringed and ribbon seals etc.
• Wildlife Migration: Each spring, one of the largest wildlife migrations on earth passes
through this narrow gateway to reach the Arctic's incredibly nutrient-rich and productive
waters.
• The settlement of the Americas began when Paleolithic hunter-gatherers entered North
America from the North Asian Mammoth steppe via the Bering Land bridge, which had
formed between northeastern Siberia and western Alaska due to the lowering of sea level
during the Last Glacial Maximum (26,000 to 19,000 years ago).

Office Complex 6, 3rd Floor, Old Rajinder Nagar New Delhi 110060|
Ph: +91-8826486658, +918826496658, | Email: students@levelupias.com
Q78.
Answer: a
Explanation:
Nursultan is the capital of Kazakhstan and
is northern most.
Bishkek (capital of Kyrgyzstan)
Tashkent (Capital of Uzbekistan)
Dushanbe (Capital of Tajikistan)
Ashgabat (capital of Turkmenistan)

Q79.
Answer: d
Explanation:

• Today, 21 countries form


border with
Mediterranean sea.
• Turkey, Syria, Lebanon,
Israel, Egypt, Cyprus,
Libya, Tunisia, Algeria,
Morocco, Spain (note: No
Portugal), France, Italy,
Malta, Slovania, Croatia,
Bosnia and Herzegovina,
Montenegro, Albania, and
Greece.

Q80.
Answer: a
Explanation:

• Sudan (Republic of Sudan) is a country in north-east Africa. It is bordered by the Central


African Republic to the Southwest, Chad to the West, Libya to the northwest, Egypt in North,
Eritrea in the north-east, Ethiopia in the southwest and South Sudan in South. So, Statement
1 is correct.
• It also borders the Red Sea on its northeast and east. It doesn't border the Mediterranean
Sea. So, Statement 2 is not correct.
• In April 2023, India launched operation Kaveri to evacuate citizens from Sudan amidst
violence. A vicious power struggle between the regular army and a powerful paramilitary
force (Rapid Support Force) has led to violence across Sudan for more than a week now. The
power struggle has seen heavy bombardment of Khartoum city.

Office Complex 6, 3rd Floor, Old Rajinder Nagar New Delhi 110060|
Ph: +91-8826486658, +918826496658, | Email: students@levelupias.com
Q81.
Answer: a
Explanation:
Changed Nature of Peasant Movements after 1857
• Peasants emerged as the main force in agrarian movements, fighting directly for their
own demands.
• The demands were centred almost wholly on economic issues.
• The movements were directed against the immediate enemies of the peasant—
foreign planters and indigenous zamindars and moneylenders.
• Statement 1 is correct: The struggles were directed towards specific and limited
objectives and redressal of particular grievances.
• Statement 2 is correct: The peasants developed a strong awareness of their legal
rights and asserted them in and outside the courts.
• Statement 3 is incorrect: Colonialism was not the target of these movements.
• It was not the objective of these movements to end the system of subordination or
exploitation of the peasants.
• Territorial reach was limited.
• There was no continuity of struggle or long-term organisation.

Q82.
Answer: d
Explanation:
There was a confrontation between moderates and extremists in the congress over the Mont-
Ford Reforms of 1919. The moderates welcomed it and the extremist opposed it which
created a schism in the congress. The moderate leaders like Surendra Nath Banarjea, Tej
Bahadur Sapru, M.R. Jayakar left the congress and formed a new organisation named Indian
National Liberation Federation in 1919.
Indian Independence League: Within the congress, leaders like Jawharlal Nehru and Subhash
Chandra Bose were dissatisfied with the Nehru report because it demended mere dominion
status not complete independence. To press their demend Jawaharlal Nehru and Subhash
Chandra Bose formed Indian Independence League in 1928.
Although the resolution affirming complete independence as the goal by the congress was
moved by Gandhi himself, and passed in 1929, Mahatma Gandhi put forward before the
Viceroy Lord Irwin, on 30 January 1930 his 11 demands:
• The repeal of the Arms Act and the abolition of the salt tax.
• The release of all political prisoners.
• Withdrawal of the controversial police regulations.
• Restoration of the confiscated lands of the peasants in the Punjab and NWFP.
• Reduction of military expenditure.
• The abolition of the CID.
• The restoration of free speech and the press.
• The establishment of a representative government in India.

Office Complex 6, 3rd Floor, Old Rajinder Nagar New Delhi 110060|
Ph: +91-8826486658, +918826496658, | Email: students@levelupias.com
• The grant of compensation to those who suffered in the civil disobedience movement.
• Release of all political prisoners who had not used violence.
• Grant of pardon to all those who had been convicted but had not used violence.
In March 1931, a special session of the Congress was held at Karachi to endorse the Gandhi-
Irwin pact. In this session only, two resolutions were adopted- one on fundamental rights and
other on the National Economic Policy which represented the party's social, economic and
political programme.

Q83.
Answer: c
Explanation:
The emergence of what became known as parallel governments in some parts of the country
was a significant feature of the Quit India Movement.
Chittu Pande, who called himself a Gandhian, formed a parallel government and captured all
the ten police stations in Ballia, in east UP in August 1942. Though it succeeded in persuading
the Collector to hand over power and release all the arrested Congress leaders, the parallel
government could not last long, and when the soldiers arrived a week later, they discovered
that the leaders had fled.
The Jatiya Sarkar: It was established on December 17, 1942, in Tamluk, Bengal's Midnapur
district, and lasted until September 1944. Tamluk was a place where Gandhi's constructive
work had made significant progress, as well as the site of previous mass struggles. The Jatiya
Sarkar helped with cyclone relief, school grants, and the formation of an armed Vidyut Vahini.
It also established arbitration courts. It could continue its operations with relative ease
because it was located in a relatively remote area. The main leaders of the Jatiya Sarkar were
Satish Chandra Samanta, Sushil Kumar Dhara, Ajoy Mukherjee and Matangini Hazra.
The Prati Sarkar: Satara, Maharashtra, became the home of the world's longest-running and
most effective parallel government. The region has been involved in the Quit India Movement
since its inception.
The most important leader was Y.B. Chavan, who had connections with Achyut Patwardhan
and other underground leaders. However, by the end of 1942, this phase had come to an end,
with nearly 2,000 people arrested.

Q84.
Answer: b
Explanation:
Statement 1 is incorrect: Mahabhashya is a detailed commentary on Panini's "Ashtadhyayi"
written by Patanjali. The Mahabhashya is one of the most important works on Sanskrit
grammar and is still widely studied today. Patanjali is also known for his work "Patanjala
Yoga Sutras," which is a text on the philosophy and practice of yoga.
Statement 2 is correct: Panini, Katyana, and Patanjali are considered as the three most
famous Sanskrit grammarians of ancient India. Panini's Ashtadhyayi is the earliest known
systematic grammar of Sanskrit, while Katyana and Patanjali wrote commentaries on Panini's
work that are also considered important works in the field of Sanskrit grammar.

Q85.
Answer: c

Office Complex 6, 3rd Floor, Old Rajinder Nagar New Delhi 110060|
Ph: +91-8826486658, +918826496658, | Email: students@levelupias.com
Explanation:
• Deula is an architectural element in a Hindu temple in the Kalinga architecture style
of the Odishan temples in Eastern India. Sometimes the whole temple is also referred
to as Deula. The word "deula" in Odia language means a building structure built with
a particular style that is seen in most of the temples from Odisha.
• It is the deula which makes three distinct types of temples in Kalinga Architecture:
Rekha Deula, Pidha Deula and Khakhara Deula.
• The former two are associated with Vishnu, Surya and Shiva temples while the third is
mainly with Chamunda and Durga temples.
• The Rekha Deula and Khakhara Deula houses the sanctum sanctorum while the Pidha
Deula constitutes outer dancing and offering halls.
• Lingaraj temple, Jaggannath temple and Konark sun temple are some important
example of Kalinga style of temple architecture.

Q86.
Answer: b
Explanation:
Pair 1 is incorrectly matched: Sringeri Sharadamba Temple: Sringeri is a popular temple
town in Karnataka. It is known for Sringeri Sharadha Peetha which was established by Adi
Shankaracharya. Interestingly, the River Tunga flows right next to the temple and fishing is
prohibited in this section of Tunga River.
Pair 2 is correctly matched: Omkareshwar Temple: Omkareshwar Temple on the banks of
river Narmada is one of the 12 Jyotirlingas in India. Omkareshwar Temple is dedicated to
Lord Shiva, and it is in Khandwa of Madhya Pradesh.
Pair 3 is correctly matched: The Brihadeeswara Temple is located on the banks of the river
Cauvery in the city of Thanjavur (Tanjore), in the Indian state of Tamil Nadu.
Pair 4 is incorrectly matched: Kedarnath Temple is situated in the Himalayan range of
Garhwal area (Uttarakhand). The Kedarnath Temple is located on the banks of the
Mandakini River in the town of Kedarnath, which is situated in the Rudraprayag district of
the Indian state of Uttarakhand.

Q87.
Answer: c
Explanation:
Trace elements are minerals that are required by the human body in very small amounts,
typically less than 100 milligrams per day. Some of the important trace elements that are
found in the human body are Iron, Zinc, Copper, Selenium, Manganese, Chromium, Iodine,
Fluorine, Cobalt and Molybdenum.
These trace elements play important roles in various physiological processes in the body,
such as enzyme activation, hormone production, and immune function. However, it is
important to note that excessive intake of some of these trace elements can be toxic, so it is
important to maintain a balanced diet and avoid taking supplements without medical
supervision.

Office Complex 6, 3rd Floor, Old Rajinder Nagar New Delhi 110060|
Ph: +91-8826486658, +918826496658, | Email: students@levelupias.com
Ilmenite is a mineral composed of iron, titanium, and other elements, and it is primarily
used in industrial applications, such as in the production of titanium dioxide.
Terbium is a chemical element that is not considered an essential micronutrient for human
health, and it is not normally found in the human body. Terbium is a rare earth metal that is
used in various industrial and technological applications.

Q88.
Answer: a
Explanation:
Torrefaction: Torrefaction is a thermal process to convert biomass into a coal-like material
which has better fuel characteristics than the original biomass.
This changes the elements of the biomass into ‘coal-like’ pellets. These pellets can be used
for combustion along with coal for industrial applications like steel and cement production.
Paddy straw made into pellets or torrefied can be mixed with coal in thermal power plants.
This saves coal as well as reduces carbon emissions.
Bio-Coal: It is commonly referred to as synthetic coal, is created through the torrefaction of
biomass. The bio-coal has similar characteristics to traditional fossil-based coal, and thus,
viable option to reduce greenhouse gas emissions.

Q89.
Answer: a
Explanation:
• Environmental DNA (eDNA) refers to genetic material that is obtained from environmental
samples, such as soil, water, or air. This genetic material comes from organisms that are
present in the environment, either through their shedding of cells, tissues, or fluids, or
through their excretion of DNA-containing waste products. So, statement 1 is correct.
• eDNA may not always provide details of all the species present in a habitat, as it depends on
factors such as the efficiency of DNA extraction and sequencing, and the sensitivity of
detection methods used. So, statement 2 is incorrect.
• eDNA can be used as a tool for detecting and identifying species in the environment,
including those that may be difficult to observe directly, such as rare or elusive species. This
is accomplished by collecting environmental samples and using molecular techniques to
isolate and sequence the DNA present in the samples.
• eDNA has numerous applications in environmental monitoring and management, including
in the detection of invasive species, monitoring of endangered species, and tracking of
pathogens and disease outbreaks. So, statement 3 is correct.

Q90.
Answer: b
Explanation:
Forest Survey of India (FSI) assessed the carbon stock (which is the amount of carbon
sequestrated from the atmosphere and stored in biomass, deadwood, soil, and litter in the
forest) in India’s forests for the first time in 2004 and then biennially since 2011. (Statement
1 is correct)
- The Indian State of Forest Report (ISFR) estimates the carbon stock of forests to be about
7,204 million tonnes in 2021 which is an increase of 79.4 million tonnes of carbon stock as

Office Complex 6, 3rd Floor, Old Rajinder Nagar New Delhi 110060|
Ph: +91-8826486658, +918826496658, | Email: students@levelupias.com
compared to the estimates of the previous assessment for 2019. This translates into carbon
emissions sequestrated through forest and tree cover to be 30.1 billion tonnes of CO2
equivalent. (Statement 3 is correct)
- Among the Indian States, Arunachal Pradesh has the maximum carbon stock in forests
(1023.84 million tonnes), followed by Madhya Pradesh (609.25 million tonnes). (Statement
2 is incorrect)

Q91.
Answer: c
Explanation
• The Sikkim Himalayas are home to some of the highest peaks in the world, including
Kangchenjunga, the third highest mountain in the world.
• The name "Kangchenjunga" means "Five Treasures of Snow," referring to the mountain's
five peaks.
• Zemu Glacier is located at Kangchenjunga Mountains, which is the source of Teesta River.
• The Sikkim Himalayas are rich in biodiversity, with a wide range of flora and fauna, including
rare and endangered species like the Red Panda, Snow Leopard, and Himalayan black bear.
• The Sikkim Himalayas are known for their stunning natural beauty, with snow-capped peaks,
alpine forests, and glacial lakes.
• The Sikkim Himalayas are an important spiritual and cultural center, with many Buddhist
monasteries and pilgrimage sites located throughout the region.
• The Sikkim Himalayas are home to a diverse range of ethnic groups, including the Lepchas,
Bhutias, and Nepalis, who have their own unique cultures and traditions.

Q92.
Answer: d
Explanation:
· Lonar Lake, also known as Lonar crater, is a saline and alkaline lake located at Lonar in
Buldhana district, Maharashtra. It is a notified National Geo-heritage Monument.
· Sultanpur national park in Gurgaon district of Haryana is a bird paradise for bird watchers.
It is famous for its migratory as well as resident birds.
· The Bakhira Bird Sanctuary is situated in Gorakhpur city. It is the largest natural flood plain
wetland in Uttar Pradesh. Bakhira Wildlife Sanctuary in UP provides a safe wintering and
staging ground for many species of the Central Asian Flyway
· Khijadiya sanctuary is in Gujarat. It is a freshwater wetland located near the coast of the
Gulf of Kutch.

Q93.
Answer: c
Explanation:
Evapotranspiration is the process by which water is taken up by plants through their roots,
transported to their leaves, and released into the atmosphere as water vapor through tiny
pores called stomata. This process involves both evaporation (of water from the soil and
plant surfaces) and transpiration (of water from the plant's stomata). Evapotranspiration

Office Complex 6, 3rd Floor, Old Rajinder Nagar New Delhi 110060|
Ph: +91-8826486658, +918826496658, | Email: students@levelupias.com
plays a crucial role in the water cycle, as it accounts for a significant portion of the water
that returns to the atmosphere and helps to regulate the Earth's climate.

Wetlands are important ecosystems that provide many ecological services, including
filtering water and providing habitat for a variety of plants and animals. Wetlands can also
play an important role in evapotranspiration, as they are typically characterized by high
water tables and dense vegetation.

Q94.
Answer: d
Explanation:
The United Nations General Assembly, with the governments of Portugal and Kenya, hosted
a high-level United Nations Ocean Conference to Support the Implementation of
Sustainable Development Goal 14 (SDG 14) in Lisbon, June 27 - July 1, 2022.
All 198 members of the UN have unanimously adopted the Lisbon Declaration on Ocean
Conservation in July 2022 on the last day of the UN Ocean Conference 2022. (Statement 1 is
incorrect and statement 2 is correct)
The Lisbon Declaration seeks to implement Sustainable Development Goal 14, which aims to
conserve and sustainably use the oceans, seas, and marine resources for development.
(Statement 3 is correct)
Participants at the conference also agreed to work on preventing, reducing and controlling
marine pollution. It includes:
• Nutrient pollution
• Untreated wastewater
• Solid waste discharges
• Hazardous substances
• Emissions from the maritime sector, including shipping, shipwrecks
Anthropogenic underwater noise

Q95.
Answer: b
Explanation:
The right to hoist a national flag with respect and dignity is protected under Fundamental
right within the meaning of Article 19(1)(a), which was held in Union of India v. Naveen
Jindal and others case.
The Part II Section I of the Flag code of India 2002 states that “There shall be no restriction
on the display of national flag by members of general public, private organizations and
educational institutions, but since it is also an explicit Fundamental right under article
19(1)(a), and we have to choose the most logical answer, the right answer should be B.

Q96.
Answer: b
Explanation: The concept of freedom refers to the state of being free from external
constraints or limitations that may prevent an individual from exercising their choices and
actions. It further includes certain conditions which are needed to develop their talents.

Office Complex 6, 3rd Floor, Old Rajinder Nagar New Delhi 110060|
Ph: +91-8826486658, +918826496658, | Email: students@levelupias.com
However, it does not mean that one has the absolute ability to do whatever they want,
regardless of its impact on others. Instead, freedom is linked with the idea of social
responsibility, and it is only meaningful if it is accompanied by a sense of respect for others
and a willingness to act in the interest of the collective good. Thus, option (b) correctly
captures the essence of freedom as a state of being free from any kind of coercion,
oppression, or exploitation and conditions in which people can develop their talents.

Q97.
Answer: d
Explanation:
Since Article 32 of the constitution guarantees the right to move the supreme court for the
enforcement of all or any of the fundamental rights and provides quick relief against the
abridgement of any fundamental rights, Dr. Ambedkar called this right called as the heart
and soul of Indian constitution.
Article 32 (1) - The right to move the Supreme Court by appropriate proceedings for the
enforcement of the rights conferred by this Part is guaranteed. Further, Article 32(4) -The
right guaranteed by this article shall not be suspended except as otherwise provided for by
this Constitution.

Q98.
Answer: d
Explanation: Secularism refers to idea of separation of religion and state, it intends to
protect the freedom of individual to exit from their religion, it also provides freedom to
interpret the religious teachings differently.
Secularism also involves providing equal opportunities to all citizens, regardless of their
religious beliefs, and respecting their rights to practice their religion without interference.
The government is expected to maintain a distance from religious affairs and not promote
any particular religion or sect.

Q99.
Answer: d
Explanation:
Statement 1 is correct- Section 2(e)(ii) of RTI Act says that the Chief Justice of India is the
“competent authority” vested with rule making powers in case of supreme court of India.
Statement 2 is correct- Section 8(1)(b) states that there shall be no obligation to give any
citizen, information which has been expressly forbidden to be published by any court of law
or tribunal or the disclosure of which may constitute contempt of court. This exemption will
only apply if a specific order of the Court or tribunal says the particular information has
been prohibited from disclosure.
Statement 3 is incorrect- The RTI act is not applicable to discussions of the Collegium,
however the final resolution or decision is required to be uploaded on the Supreme court
website.
This provision gives the authority only to competent authorities to frame rules. Primarily,
rules can provide for application fees, additional fees for providing information, formats for
RTI applications and appeals. It is important to note that this power can only be exercised by
competent authorities and not by public authorities.

Office Complex 6, 3rd Floor, Old Rajinder Nagar New Delhi 110060|
Ph: +91-8826486658, +918826496658, | Email: students@levelupias.com
Additional Information: - In a 2019 judgement, a five-judge constitution bench led by CJI
Ranjan Gogoi declared that office of CJI is a ‘public authority’ under the RTI Act.

Q100.
Answer: c
Explanation: Both the statements are correct.
The term “executive power” is not defined in the constitution. Article 73, however provides
that the executive power of the Union shall extend to the matters with respect to which
parliament has powers to make laws.
Article 61 lays down the detailed procedure for impeachment of president and it is
mentioned below:
61. Procedure for impeachment of the President
(1) When a President is to be impeached for violation of the Constitution, the charge shall
be preferred by either House of Parliament
(2) No such charge shall be preferred unless-
(a) the proposal to prefer such charge is contained in a resolution which has been moved
after at least fourteen days' notice in writing signed by not less than one fourth of the total
number of members of the House has been given of their intention to move the resolution,
and
(b) such resolution has been passed by a majority of not less than two thirds of the total
membership of the House.
(3) When a charge has been so preferred by either House of Parliament, the other House
shall investigate the charge or cause the charge to be investigated and the President shall
have the right to appear and to be represented as such investigation.
(4) If as a result of the investigation a resolution is passed by a majority of not less than two
thirds of the total membership of the House by which the charge was investigated or cause
to be investigated, declaring that the charge preferred against the President has been
sustained, such resolution shall have the effect of removing the President from his office as
from the date on which the resolution is so passed.

Office Complex 6, 3rd Floor, Old Rajinder Nagar New Delhi 110060|
Ph: +91-8826486658, +918826496658, | Email: students@levelupias.com

You might also like